How to you prove $|a|=|-a|$ is true












1












$begingroup$


Let $a in R$ prove that:



$|a|=|-a|$



I am new to proofs so this is my attempt:



Case 1: $|a|=|-a|$



$$(a)=-(-a)$$



$$a=a$$



Case 2: $|-a|=|-a|$



$$-(-a)=-(-a)$$



$$a=a$$



Case 3: $|a|=|a|$



$$a=a$$



Is this the correct way to approach a proof like this?










share|cite|improve this question











$endgroup$








  • 1




    $begingroup$
    I don't understand the cases you are distinguishing.
    $endgroup$
    – lulu
    Jan 18 at 2:23






  • 3




    $begingroup$
    Make your cases $a>0, a<0, a=0$ An alternative would be to say $|a| = sqrt {a^2} = sqrt {(-a)^2} = |-a|$
    $endgroup$
    – Doug M
    Jan 18 at 2:23








  • 1




    $begingroup$
    Hint: $|a|$ is the greater of $a,-a$.
    $endgroup$
    – lulu
    Jan 18 at 2:24






  • 2




    $begingroup$
    It's going to depend on how you define $|a|$. There are several ways to do it; which one are you using?
    $endgroup$
    – dbx
    Jan 18 at 2:25
















1












$begingroup$


Let $a in R$ prove that:



$|a|=|-a|$



I am new to proofs so this is my attempt:



Case 1: $|a|=|-a|$



$$(a)=-(-a)$$



$$a=a$$



Case 2: $|-a|=|-a|$



$$-(-a)=-(-a)$$



$$a=a$$



Case 3: $|a|=|a|$



$$a=a$$



Is this the correct way to approach a proof like this?










share|cite|improve this question











$endgroup$








  • 1




    $begingroup$
    I don't understand the cases you are distinguishing.
    $endgroup$
    – lulu
    Jan 18 at 2:23






  • 3




    $begingroup$
    Make your cases $a>0, a<0, a=0$ An alternative would be to say $|a| = sqrt {a^2} = sqrt {(-a)^2} = |-a|$
    $endgroup$
    – Doug M
    Jan 18 at 2:23








  • 1




    $begingroup$
    Hint: $|a|$ is the greater of $a,-a$.
    $endgroup$
    – lulu
    Jan 18 at 2:24






  • 2




    $begingroup$
    It's going to depend on how you define $|a|$. There are several ways to do it; which one are you using?
    $endgroup$
    – dbx
    Jan 18 at 2:25














1












1








1





$begingroup$


Let $a in R$ prove that:



$|a|=|-a|$



I am new to proofs so this is my attempt:



Case 1: $|a|=|-a|$



$$(a)=-(-a)$$



$$a=a$$



Case 2: $|-a|=|-a|$



$$-(-a)=-(-a)$$



$$a=a$$



Case 3: $|a|=|a|$



$$a=a$$



Is this the correct way to approach a proof like this?










share|cite|improve this question











$endgroup$




Let $a in R$ prove that:



$|a|=|-a|$



I am new to proofs so this is my attempt:



Case 1: $|a|=|-a|$



$$(a)=-(-a)$$



$$a=a$$



Case 2: $|-a|=|-a|$



$$-(-a)=-(-a)$$



$$a=a$$



Case 3: $|a|=|a|$



$$a=a$$



Is this the correct way to approach a proof like this?







proof-verification proof-writing proof-explanation absolute-value






share|cite|improve this question















share|cite|improve this question













share|cite|improve this question




share|cite|improve this question








edited Jan 18 at 7:31









Michael Rozenberg

103k1891195




103k1891195










asked Jan 18 at 2:21









JohnJohn

325




325








  • 1




    $begingroup$
    I don't understand the cases you are distinguishing.
    $endgroup$
    – lulu
    Jan 18 at 2:23






  • 3




    $begingroup$
    Make your cases $a>0, a<0, a=0$ An alternative would be to say $|a| = sqrt {a^2} = sqrt {(-a)^2} = |-a|$
    $endgroup$
    – Doug M
    Jan 18 at 2:23








  • 1




    $begingroup$
    Hint: $|a|$ is the greater of $a,-a$.
    $endgroup$
    – lulu
    Jan 18 at 2:24






  • 2




    $begingroup$
    It's going to depend on how you define $|a|$. There are several ways to do it; which one are you using?
    $endgroup$
    – dbx
    Jan 18 at 2:25














  • 1




    $begingroup$
    I don't understand the cases you are distinguishing.
    $endgroup$
    – lulu
    Jan 18 at 2:23






  • 3




    $begingroup$
    Make your cases $a>0, a<0, a=0$ An alternative would be to say $|a| = sqrt {a^2} = sqrt {(-a)^2} = |-a|$
    $endgroup$
    – Doug M
    Jan 18 at 2:23








  • 1




    $begingroup$
    Hint: $|a|$ is the greater of $a,-a$.
    $endgroup$
    – lulu
    Jan 18 at 2:24






  • 2




    $begingroup$
    It's going to depend on how you define $|a|$. There are several ways to do it; which one are you using?
    $endgroup$
    – dbx
    Jan 18 at 2:25








1




1




$begingroup$
I don't understand the cases you are distinguishing.
$endgroup$
– lulu
Jan 18 at 2:23




$begingroup$
I don't understand the cases you are distinguishing.
$endgroup$
– lulu
Jan 18 at 2:23




3




3




$begingroup$
Make your cases $a>0, a<0, a=0$ An alternative would be to say $|a| = sqrt {a^2} = sqrt {(-a)^2} = |-a|$
$endgroup$
– Doug M
Jan 18 at 2:23






$begingroup$
Make your cases $a>0, a<0, a=0$ An alternative would be to say $|a| = sqrt {a^2} = sqrt {(-a)^2} = |-a|$
$endgroup$
– Doug M
Jan 18 at 2:23






1




1




$begingroup$
Hint: $|a|$ is the greater of $a,-a$.
$endgroup$
– lulu
Jan 18 at 2:24




$begingroup$
Hint: $|a|$ is the greater of $a,-a$.
$endgroup$
– lulu
Jan 18 at 2:24




2




2




$begingroup$
It's going to depend on how you define $|a|$. There are several ways to do it; which one are you using?
$endgroup$
– dbx
Jan 18 at 2:25




$begingroup$
It's going to depend on how you define $|a|$. There are several ways to do it; which one are you using?
$endgroup$
– dbx
Jan 18 at 2:25










3 Answers
3






active

oldest

votes


















1












$begingroup$

Because the distances from $a$ and from $-a$ to the zero are equal.






share|cite|improve this answer









$endgroup$





















    0












    $begingroup$

    The definition of absolute value of a real number $a$ is $|a|=sqrt{x^2}$. Since $a^2=(-a)^2$ and square roots only take positive values $|a|=|-a|$






    share|cite|improve this answer









    $endgroup$





















      0












      $begingroup$

      If you are going to do a case by case proof, you only need to consider two cases:



      Case 1:
      $a ge 0 \ Rightarrow |a|=a, |-a|=a \ Rightarrow |a|=|-a|$



      Case 2:
      $a < 0 \ Rightarrow |a|=-a, |-a|=-a \ Rightarrow |a|=|-a|$






      share|cite|improve this answer









      $endgroup$













        Your Answer





        StackExchange.ifUsing("editor", function () {
        return StackExchange.using("mathjaxEditing", function () {
        StackExchange.MarkdownEditor.creationCallbacks.add(function (editor, postfix) {
        StackExchange.mathjaxEditing.prepareWmdForMathJax(editor, postfix, [["$", "$"], ["\\(","\\)"]]);
        });
        });
        }, "mathjax-editing");

        StackExchange.ready(function() {
        var channelOptions = {
        tags: "".split(" "),
        id: "69"
        };
        initTagRenderer("".split(" "), "".split(" "), channelOptions);

        StackExchange.using("externalEditor", function() {
        // Have to fire editor after snippets, if snippets enabled
        if (StackExchange.settings.snippets.snippetsEnabled) {
        StackExchange.using("snippets", function() {
        createEditor();
        });
        }
        else {
        createEditor();
        }
        });

        function createEditor() {
        StackExchange.prepareEditor({
        heartbeatType: 'answer',
        autoActivateHeartbeat: false,
        convertImagesToLinks: true,
        noModals: true,
        showLowRepImageUploadWarning: true,
        reputationToPostImages: 10,
        bindNavPrevention: true,
        postfix: "",
        imageUploader: {
        brandingHtml: "Powered by u003ca class="icon-imgur-white" href="https://imgur.com/"u003eu003c/au003e",
        contentPolicyHtml: "User contributions licensed under u003ca href="https://creativecommons.org/licenses/by-sa/3.0/"u003ecc by-sa 3.0 with attribution requiredu003c/au003e u003ca href="https://stackoverflow.com/legal/content-policy"u003e(content policy)u003c/au003e",
        allowUrls: true
        },
        noCode: true, onDemand: true,
        discardSelector: ".discard-answer"
        ,immediatelyShowMarkdownHelp:true
        });


        }
        });














        draft saved

        draft discarded


















        StackExchange.ready(
        function () {
        StackExchange.openid.initPostLogin('.new-post-login', 'https%3a%2f%2fmath.stackexchange.com%2fquestions%2f3077764%2fhow-to-you-prove-a-a-is-true%23new-answer', 'question_page');
        }
        );

        Post as a guest















        Required, but never shown

























        3 Answers
        3






        active

        oldest

        votes








        3 Answers
        3






        active

        oldest

        votes









        active

        oldest

        votes






        active

        oldest

        votes









        1












        $begingroup$

        Because the distances from $a$ and from $-a$ to the zero are equal.






        share|cite|improve this answer









        $endgroup$


















          1












          $begingroup$

          Because the distances from $a$ and from $-a$ to the zero are equal.






          share|cite|improve this answer









          $endgroup$
















            1












            1








            1





            $begingroup$

            Because the distances from $a$ and from $-a$ to the zero are equal.






            share|cite|improve this answer









            $endgroup$



            Because the distances from $a$ and from $-a$ to the zero are equal.







            share|cite|improve this answer












            share|cite|improve this answer



            share|cite|improve this answer










            answered Jan 18 at 7:21









            Michael RozenbergMichael Rozenberg

            103k1891195




            103k1891195























                0












                $begingroup$

                The definition of absolute value of a real number $a$ is $|a|=sqrt{x^2}$. Since $a^2=(-a)^2$ and square roots only take positive values $|a|=|-a|$






                share|cite|improve this answer









                $endgroup$


















                  0












                  $begingroup$

                  The definition of absolute value of a real number $a$ is $|a|=sqrt{x^2}$. Since $a^2=(-a)^2$ and square roots only take positive values $|a|=|-a|$






                  share|cite|improve this answer









                  $endgroup$
















                    0












                    0








                    0





                    $begingroup$

                    The definition of absolute value of a real number $a$ is $|a|=sqrt{x^2}$. Since $a^2=(-a)^2$ and square roots only take positive values $|a|=|-a|$






                    share|cite|improve this answer









                    $endgroup$



                    The definition of absolute value of a real number $a$ is $|a|=sqrt{x^2}$. Since $a^2=(-a)^2$ and square roots only take positive values $|a|=|-a|$







                    share|cite|improve this answer












                    share|cite|improve this answer



                    share|cite|improve this answer










                    answered Jan 18 at 4:41









                    Juan123Juan123

                    948




                    948























                        0












                        $begingroup$

                        If you are going to do a case by case proof, you only need to consider two cases:



                        Case 1:
                        $a ge 0 \ Rightarrow |a|=a, |-a|=a \ Rightarrow |a|=|-a|$



                        Case 2:
                        $a < 0 \ Rightarrow |a|=-a, |-a|=-a \ Rightarrow |a|=|-a|$






                        share|cite|improve this answer









                        $endgroup$


















                          0












                          $begingroup$

                          If you are going to do a case by case proof, you only need to consider two cases:



                          Case 1:
                          $a ge 0 \ Rightarrow |a|=a, |-a|=a \ Rightarrow |a|=|-a|$



                          Case 2:
                          $a < 0 \ Rightarrow |a|=-a, |-a|=-a \ Rightarrow |a|=|-a|$






                          share|cite|improve this answer









                          $endgroup$
















                            0












                            0








                            0





                            $begingroup$

                            If you are going to do a case by case proof, you only need to consider two cases:



                            Case 1:
                            $a ge 0 \ Rightarrow |a|=a, |-a|=a \ Rightarrow |a|=|-a|$



                            Case 2:
                            $a < 0 \ Rightarrow |a|=-a, |-a|=-a \ Rightarrow |a|=|-a|$






                            share|cite|improve this answer









                            $endgroup$



                            If you are going to do a case by case proof, you only need to consider two cases:



                            Case 1:
                            $a ge 0 \ Rightarrow |a|=a, |-a|=a \ Rightarrow |a|=|-a|$



                            Case 2:
                            $a < 0 \ Rightarrow |a|=-a, |-a|=-a \ Rightarrow |a|=|-a|$







                            share|cite|improve this answer












                            share|cite|improve this answer



                            share|cite|improve this answer










                            answered Jan 18 at 10:03









                            gandalf61gandalf61

                            8,719725




                            8,719725






























                                draft saved

                                draft discarded




















































                                Thanks for contributing an answer to Mathematics Stack Exchange!


                                • Please be sure to answer the question. Provide details and share your research!

                                But avoid



                                • Asking for help, clarification, or responding to other answers.

                                • Making statements based on opinion; back them up with references or personal experience.


                                Use MathJax to format equations. MathJax reference.


                                To learn more, see our tips on writing great answers.




                                draft saved


                                draft discarded














                                StackExchange.ready(
                                function () {
                                StackExchange.openid.initPostLogin('.new-post-login', 'https%3a%2f%2fmath.stackexchange.com%2fquestions%2f3077764%2fhow-to-you-prove-a-a-is-true%23new-answer', 'question_page');
                                }
                                );

                                Post as a guest















                                Required, but never shown





















































                                Required, but never shown














                                Required, but never shown












                                Required, but never shown







                                Required, but never shown

































                                Required, but never shown














                                Required, but never shown












                                Required, but never shown







                                Required, but never shown







                                Popular posts from this blog

                                Mario Kart Wii

                                What does “Dominus providebit” mean?

                                Antonio Litta Visconti Arese